100% satisfaction guarantee Immediately available after payment Both online and in PDF No strings attached
logo-home
Solution Manual for Game Theory Basics 1st Edition By Bernhard von Stengel, ISBN: 9781108843300, All 12 Chapters Covered $14.49   Add to cart

Exam (elaborations)

Solution Manual for Game Theory Basics 1st Edition By Bernhard von Stengel, ISBN: 9781108843300, All 12 Chapters Covered

 19 views  0 purchase
  • Course
  • Game Theory Basics By Bernhard Von Stengel
  • Institution
  • Game Theory Basics By Bernhard Von Stengel

Solution Manual for Game Theory Basics 1st Edition By Bernhard von Stengel, ISBN: 9781108843300, All 12 Chapters Covered, Verified Latest Edition Solution Manual for Game Theory Basics 1st Edition By Bernhard von Stengel, ISBN: 9781108843300, All 12 Chapters Covered, Verified Latest Edition Test ba...

[Show more]

Preview 4 out of 67  pages

  • September 13, 2024
  • 67
  • 2024/2025
  • Exam (elaborations)
  • Questions & answers
book image

Book Title:

Author(s):

  • Edition:
  • ISBN:
  • Edition:
  • Game Theory Basics By Bernhard Von Stengel
  • Game Theory Basics By Bernhard Von Stengel
avatar-seller
reagandave
SOLUTION MANUAL
Game Theory Basics 1st Edition
By Bernhard von Stengel. Chapters 1 - 12




1

,TABLE OF CONTENTS

1 - Nim and Combinatorial Games

2 - Congestion Games

3 - Games in Strategic Form

4 - Game Trees with Perfect Information

5 - Expected Utility

6 - Mixed Equilibrium

7 - Brouwer’s Fixed-Point Theorem

8 - Zero-Sum Games

9 - Geometry of Equilibria in Bimatrix Games

10 - Game Trees with Imperfect Information

11 - Bargaining

12 - Correlated Equilibrium




2

,Game Theory Basics
Solutions to Exercises
© Bernhard von Stengel 2022

Solution to Exercise 1.1

(a) Let ≤ be defined by (1.7). To show that ≤ is transitive, consider x, y, z with x ≤ y and y ≤ z. If
x = y then x ≤ z, and if y = z then also x ≤ z. So the only case left is x < y and y < z,
which implies x < z because < is transitive, and hence x ≤ z.
Clearly, ≤ is reflexive because x = x and therefore x ≤ x.
To show that ≤is antisymmetric, consider x and y with x y and
≤ y x. If≤ we had x ≠ y
then x < y and y < x, and by transitivity x < x which contradicts (1.38). Hence x = y, as
required. This shows that ≤ is a partial order.
Finally, we show (1.6), so we have to show that x < y implies x y and x≤ ≠ y and vice versa.
Let x < y, which implies x y by (1.7). If we had x = y then x < x, contradicting (1.38), so we
also have x ≠ y. Conversely, x y and x ≠ y ≤ imply by (1.7) x < y or x = y where the second
case is excluded, hence x < y, as required. ≤
(b) Consider a partial order and ≤ assume (1.6) as a definition of <. To show that < is transitive,
suppose x < y, that is, x y and x ≠ ≤y, and y < z, that is, y z and y ≠ z.≤ Because is
transitive, x z. If we had x = z then x y and y x and hence x = y by antisymmetry of

, which contradicts x ≠ y, so≤we have x z and x ≠ z, that≤is,x < z by≤ (1.6), as required.
≤ ≤
Also, < is irreflexive, because x < x would by definition mean x x and≤x ≠ x, but the latter
is not true.
Finally, we show (1.7), so we have to show that x ≤ y implies x < y or x = y and vice versa,
given that < is defined by (1.6). Let x ≤ y. Then if x = y, we are done, otherwise x ≠ y and
then by definition x < y. Hence, x ≤ y implies x < y or x = y. Conversely, suppose x < y or
x = y. If x < y then x ≤ y by (1.6), and if x = y then x ≤ y because ≤ is reflexive. This
completes the proof.

Solution to Exercise 1.2

(a) In analysing the games of three Nim heaps where one heap has size one, we first look at some
examples, and then use mathematical induction to prove what we conjecture to be the losing
positions. A losing position is one where every move is to a winning position, because then the
opponent will win. The point of this exercise is to formulate a precise statement to be proved,
and then to prove it.
First, if there are only two heaps recall that they are losing if and only if the heaps are of
equal size. If they are of unequal size, then the winning move is to reduce the larger heap so
that both heaps have equal size.




3

, Consider three heaps of sizes 1, m, n, where 1 m≤ n. ≤ We observe the following: 1, 1, m
is winning, by moving to 1, 1, 0. Similarly, 1, m, m is winning, by moving to 0, m, m. Next, 1,
2, 3 is losing (observed earlier in the lecture), and hence 1, 2, n for n 4 is winning. 1, 3, n is
winning for any n 3 by moving to 1, 3, 2. For 1, 4, 5, reducing any heap produces a winning

position, so this is losing. ≥
The general pattern for the losing positions thus seems to be: 1, m, m 1, for even+ numbers m.
This includes also the case m = 0, which we can take as the base case for an induction. We
now proceed to prove this formally.
First we show that if the positions of the form 1, m, n with m n are≤ losing when m is even
and n = m 1, then these are the only losing positions because any other position 1, m, n with
m n is winning. Namely, + if m = n then a winning move from 1, m, m is to 0, m, m, so we can
assume m < n. If m is even ≤then n > m 1 (otherwise we would be in the position 1, m, m 1)
and so the winning move is to 1, m, m 1. If m is odd then the winning move is + to 1, m, m 1, the
same as position 1, m 1, m (this would also+ be a winning move from 1, m, m so there the + winning
move is not unique).
– −
Second, we show that any move from 1, m, m + 1 with even m is to a winning position, using as
inductive hypothesis that 1, mJ, mJ + 1 for even mJ and mJ < m is a losing position. The move
to 0, m, m + 1 produces a winning position with counter-move to 0, m, m. A move to 1, mJ, m
+ 1 for mJ < m is to a winning position with the counter-move to 1, mJ, mJ + 1 if mJ is even and
to 1, mJ, mJ − 1 if mJ is odd. A move to 1, m, m is to a winning position with counter-move to 0,
m, m. A move to 1, m, mJ with mJ < m is also to a winning position with the counter-move to 1, mJ
− 1, mJ if mJ is odd, and to 1, mJ 1, mJ if mJ is even (in which case mJ 1 < m because m is
even). This concludes the induction proof.
This result is in agreement with the theorem on Nim heap sizes represented as sums of powers of
2: 1 m n is losing+if and only if, except for 20, the powers of + 2 making up m and n come in
∗ +∗ +∗
pairs. So these must be the same powers of 2, except for 1 = 20, which occurs in only m or n,
where we have assumed that n is the larger number, so 1 appears in the representation of n:
We have m = 2a 2b 2c for a > b > c > 1,
so m is even, and, with the same a, b, c, . . ., n = 2a+ 2b+ 2c+ · · · 1 = m 1.· · Then
·≥
∗1 + ∗ m + ∗ n ≡ ∗ 0. The following is an example using the + bit+ representation
+···+ +
where
m = 12 (which determines the bit pattern 1100, which of course depends on m):
1 = 000
1
12 = 110
0
13 = 110
1
Nim-sum 0 = 000
0

(b) We use (a). Clearly, 1, 2, 3 is losing as shown in (1.2), and because the Nim-sum of the
binary representations 01, 10, 11 is 00. Examples show that any other position is winning.
The three numbers are n, n 1, n 2. If n is even then reducing the heap of size n 2 to 1 creates
+
the position n, n 1, 1 which is losing as shown in+(a). If n is odd, then n 1 is even and n 2 =
n 1 1 + so by the same argument, a winning + move is to reduce the Nim heap of size n to 1
+ if n > 1).
(which only works + ( + )+




4

The benefits of buying summaries with Stuvia:

Guaranteed quality through customer reviews

Guaranteed quality through customer reviews

Stuvia customers have reviewed more than 700,000 summaries. This how you know that you are buying the best documents.

Quick and easy check-out

Quick and easy check-out

You can quickly pay through credit card or Stuvia-credit for the summaries. There is no membership needed.

Focus on what matters

Focus on what matters

Your fellow students write the study notes themselves, which is why the documents are always reliable and up-to-date. This ensures you quickly get to the core!

Frequently asked questions

What do I get when I buy this document?

You get a PDF, available immediately after your purchase. The purchased document is accessible anytime, anywhere and indefinitely through your profile.

Satisfaction guarantee: how does it work?

Our satisfaction guarantee ensures that you always find a study document that suits you well. You fill out a form, and our customer service team takes care of the rest.

Who am I buying these notes from?

Stuvia is a marketplace, so you are not buying this document from us, but from seller reagandave. Stuvia facilitates payment to the seller.

Will I be stuck with a subscription?

No, you only buy these notes for $14.49. You're not tied to anything after your purchase.

Can Stuvia be trusted?

4.6 stars on Google & Trustpilot (+1000 reviews)

83637 documents were sold in the last 30 days

Founded in 2010, the go-to place to buy study notes for 14 years now

Start selling
$14.49
  • (0)
  Add to cart